Every subsequence converges to L implies a_n -> L

  • Thread starter Thread starter Mr Davis 97
  • Start date Start date
  • Tags Tags
    Subsequence
Click For Summary
SUMMARY

For a bounded sequence ##\{a_n\}## where every convergent subsequence has a limit ##L##, it is proven that ##\lim_{n\to\infty}a_n = L##. The proof utilizes the concepts of ##\liminf_{n \to \infty} a_n## and ##\limsup_{n \to \infty} a_n##. Since the set of subsequential limits is solely ##\{L\}##, it follows that both ##\limsup a_n## and ##\liminf a_n## equal ##L##, confirming the limit of the sequence itself is ##L##.

PREREQUISITES
  • Understanding of bounded sequences in real analysis
  • Knowledge of subsequences and their limits
  • Familiarity with the concepts of ##\liminf## and ##\limsup##
  • Basic principles of convergence in sequences
NEXT STEPS
  • Study the properties of bounded sequences in real analysis
  • Learn about the definitions and applications of ##\liminf## and ##\limsup##
  • Explore the concept of subsequential limits and their significance
  • Review proofs involving convergence of sequences and subsequences
USEFUL FOR

Mathematics students, particularly those studying real analysis, as well as educators and anyone interested in the convergence properties of sequences.

Mr Davis 97
Messages
1,461
Reaction score
44

Homework Statement


Let ##\{a_n\}## be a bounded sequence such that every convergent subsequence has limit ##L##. Prove that ##\lim_{n\to\infty}a_n = L##.

Homework Equations

The Attempt at a Solution


I'm not really understanding this problem. Isn't ##\{a_n\}## a subsequence of itself? So isn't it immediately the case that ##\lim_{n\to\infty}a_n = L## by the hypothesis?

EDIT: Nevermind. I notice now that it says every convergent subsequence.
 
Physics news on Phys.org
Hint: This is easy considering ##\liminf_{n \to \infty}a_n## and ## \limsup_{n \to \infty} a_n##
 
Math_QED said:
Hint: This is easy considering ##\liminf_{n \to \infty}a_n## and ## \limsup_{n \to \infty} a_n##
Could we just say that since the set of subsequential limits is just ##\{L\}##, we have that ##\limsup a_n = \sup \{L\} = L = \inf \{L\} = \liminf a_n##. So ##\lim_{n\to\infty} a_n = L##?
 
Mr Davis 97 said:
Could we just say that since the set of subsequential limits is just ##\{L\}##, we have that ##\limsup a_n = \sup \{L\} = L = \inf \{L\} = \liminf a_n##. So ##\lim_{n\to\infty} a_n = L##?

Exactly. Well done!
 
Question: A clock's minute hand has length 4 and its hour hand has length 3. What is the distance between the tips at the moment when it is increasing most rapidly?(Putnam Exam Question) Answer: Making assumption that both the hands moves at constant angular velocities, the answer is ## \sqrt{7} .## But don't you think this assumption is somewhat doubtful and wrong?

Similar threads

Replies
4
Views
2K
Replies
6
Views
3K
  • · Replies 4 ·
Replies
4
Views
2K
  • · Replies 7 ·
Replies
7
Views
2K
  • · Replies 2 ·
Replies
2
Views
2K
  • · Replies 2 ·
Replies
2
Views
2K
  • · Replies 1 ·
Replies
1
Views
2K
  • · Replies 34 ·
2
Replies
34
Views
3K
  • · Replies 1 ·
Replies
1
Views
1K
  • · Replies 3 ·
Replies
3
Views
1K